Difference between revisions of "2006 AMC 10A Problems/Problem 4"

(See Also)
(Redirect)
 
Line 1: Line 1:
== Problem ==
+
#redirect [[2006 AMC 12A Problems/Problem 4]]
A digital watch displays hours and minutes with AM and PM.  What is the largest possible sum of the digits in the display?
 
 
 
<math> \mathrm{(A) \ } 17\qquad \mathrm{(B) \ } 19\qquad \mathrm{(C) \ } 21\qquad \mathrm{(D) \ } 22\qquad \mathrm{(E) \ } 23 </math>
 
 
 
== Solution ==
 
The time with largest sum is 9:59.
 
<center><math>9+5+9=23,(E)</math></center>
 
 
 
== See also ==
 
{{AMC10 box|year=2006|ab=A|num-b=3|num-a=5}}
 
 
 
[[Category:Introductory Number Theory Problems]]
 

Latest revision as of 00:05, 28 April 2008